Last visit was: 25 Apr 2024, 14:15 It is currently 25 Apr 2024, 14:15

Close

GRE Prep Club Daily Prep

Thank you for using the timer - this advanced tool can estimate your performance and suggest more practice questions. We have subscribed you to Daily Prep Questions via email.

Customized
for You

we will pick new questions that match your level based on your Timer History

Track
Your Progress

every week, we’ll send you an estimated GRE score based on your performance

Practice
Pays

we will pick new questions that match your level based on your Timer History

Not interested in getting valuable practice questions and articles delivered to your email? No problem, unsubscribe here.

Close

Request Expert Reply

Confirm Cancel
SORT BY:
Date
GRE Prep Club Team Member
Joined: 20 Feb 2017
Posts: 2506
Own Kudos [?]: 3249 [6]
Given Kudos: 1051
GPA: 3.39
Send PM
Most Helpful Community Reply
Retired Moderator
Joined: 09 Jan 2021
Posts: 576
Own Kudos [?]: 837 [5]
Given Kudos: 194
GRE 1: Q167 V156
GPA: 4
WE:Analyst (Investment Banking)
Send PM
General Discussion
Intern
Intern
Joined: 07 Apr 2021
Posts: 39
Own Kudos [?]: 11 [1]
Given Kudos: 12
Send PM
Retired Moderator
Joined: 09 Jan 2021
Posts: 576
Own Kudos [?]: 837 [2]
Given Kudos: 194
GRE 1: Q167 V156
GPA: 4
WE:Analyst (Investment Banking)
Send PM
An industry analyst asserted in his recent report that the relative [#permalink]
2
Jazzy007 wrote:
Ks1859 wrote:
Hi!

Scarcity of housing in a particular market ---> larger than normal increases in price
And then the author states example to support his point.

Which of the following is an assumption that supports the analyst's assertion?

A. In the housing market, there generally must be at least five buyers per seller in order to cause larger than normal increases in price.--> Out of scope

B. Increases in demand often reflect an influx of new buyers into the marketplace or an unusual increase in buying power on the part of the customer.-->Out of scope, nothing about new buyer is discussed

C. The U.S. housing market showed a larger than average increase in the 1990s across the country, not just in crowded urban areas.-->Irrelevant

D. Price increases do not cause people to withhold their houses from the market in the hopes that prices will increase even further in the future.--> This is the only supporting option because, if this is false, this would mean that it is not the high population but the people who are causing the scarcity.

E. A significant rise in housing prices in a specific area may cause some potential buyers to relocate to other, less pricey areas.-->Even if does, there are other buyers and the demand for the house might still rise- OUT

For assumption questions, the correct answer choice will fit one of the below:
*Eliminate an alternate cause for the stated effect
*Show that when the cause occurs, the effect occurs
*Shows that when the cause does not occur, the effect does not occur
*Eliminates the possibility that the stated relationship is reversed- Claimed effect is actually the cause of claimed cause
*Shows that the data used to make the causal statement are accurate or eliminates possible problems with the data


For this question we can use the first choice.

There might be other fit, but these are few of the important ones.

Hope this helps!




It looks to me an explanation after seeing the answer that is forcefully made to fit the answer.
As you have discarded B on the same basis I can discard D, because no where it has been talked about withholding the houses.
The passage only talks about price and population.

Really getting an answer is like luck.


Hi There!

If you know what questions actually are getting an answer is no luck. Secondly, it important to know as to why same basis and what actually is out of scope. This is not just a game where I discard B and you discard D :lol: . If you discard D ETS will discard your 1 point and you never know that one point might be so precious.


Let me try helping you to negate B and reach to D as an answer.

If I am talking about apples, the assumption is also more likely to be about apples and not oranges. The author is talking about the rise in prices of the house so, I need an assumption which talks about the prices and not something which is totally different like a buyer or new buyer.

Now, Proving D Correct, in order to see whether the option is correct add/remove the negative. i.e. in the above; if I say, Price increases did cause people to withhold their houses from the market in the hopes that prices will increase even further in the future. So in this case it would mean that the authors conclusion is wrong and as there were few houses, this cause the scarcity rather than the high population. This is what we want.

Try negating an option to solve an assumption question.

Let me know if you further feel like discarding another option :lol:

Hope this helps!
User avatar
VP
VP
Joined: 07 Jan 2021
Posts: 1463
Own Kudos [?]: 51 [0]
Given Kudos: 0
Send PM
Re: An industry analyst asserted in his recent report that the relative [#permalink]
Hello from the GRE Prep Club VerbalBot!

Thanks to another GRE Prep Club member, I have just discovered this valuable topic, yet it had no discussion for over a year. I am now bumping it up - doing my job. I think you may find it valuable (esp those replies with Kudos).

Want to see all other topics I dig out? Follow me (click follow button on profile). You will receive a summary of all topics I bump in your profile area as well as via email.
Prep Club for GRE Bot
[#permalink]
Moderators:
GRE Instructor
218 posts
GRE Instructor
1029 posts

Powered by phpBB © phpBB Group | Emoji artwork provided by EmojiOne